Integral of greatest integer function and its graph

Click For Summary
The discussion focuses on calculating the area under the graph of the greatest integer function from 1/n to 1, highlighting the jumps at specific points like 1/2, 1/3, and 1/4. Participants acknowledge the visual representation of the function, despite some glitches in the vertical lines. The importance of understanding the behavior of the function at these discontinuities is emphasized for accurate area calculation. The shared image aids in visualizing the function's characteristics. Overall, the conversation centers on effectively analyzing the integral of the greatest integer function.
tensaiyan
Messages
2
Reaction score
0
Homework Statement
Calculus problem.

How to calculate the integral of greatest function y=[1/x] ? And can someone please show me how to draw the graph of that function . Upper limit= 1,lower limit= 1/n (where n is natural number)

Please give me some hints or explanations for this kind of integral.
Relevant Equations
I already tried to answer the question but don’t know whether the answer is right or not. I attach some of my steps done below.
246770
246771
 
Physics news on Phys.org
I won't try to read a sideways picture, but here's the picture you asked for:
jumps.jpg

showing jumps at ##\frac 1 2,~ \frac 1 3,~ \frac 1 4## etc. Ignore the glitches in the vertical lines. You just need to calculate the area under the graph from ##\frac 1 n## to ##1##.
 
  • Like
Likes tensaiyan
LCKurtz said:
I won't try to read a sideways picture, but here's the picture you asked for:
View attachment 246783
showing jumps at ##\frac 1 2,~ \frac 1 3,~ \frac 1 4## etc. Ignore the glitches in the vertical lines. You just need to calculate the area under the graph from ##\frac 1 n## to ##1##.
Thanks,it was very really helpful!
 
Question: A clock's minute hand has length 4 and its hour hand has length 3. What is the distance between the tips at the moment when it is increasing most rapidly?(Putnam Exam Question) Answer: Making assumption that both the hands moves at constant angular velocities, the answer is ## \sqrt{7} .## But don't you think this assumption is somewhat doubtful and wrong?

Similar threads

Replies
5
Views
2K
  • · Replies 6 ·
Replies
6
Views
1K
  • · Replies 15 ·
Replies
15
Views
2K
  • · Replies 5 ·
Replies
5
Views
1K
  • · Replies 2 ·
Replies
2
Views
1K
Replies
2
Views
1K
  • · Replies 5 ·
Replies
5
Views
1K
  • · Replies 4 ·
Replies
4
Views
2K
Replies
3
Views
760
  • · Replies 4 ·
Replies
4
Views
2K